Proof about ordering-preserving function











up vote
0
down vote

favorite












Let $f: C_1 to C_2$ be an order preserving function. Assume that for $A subset C_1$ there exist $Sup(A) in C_1$ and $Sup(f(A)) in C_2$. Prove that $Sup(f(A)) leq f(Sup(A))$.



The statement seems obvious, but how would one go about proving this?










share|cite|improve this question






















  • Show (using the "order-preserving" assumption) that $f(sup(A))$ is an upper bound for $f(A)$. Then use that $sup(f(A))$ is the least upper bound of the same set $f(A)$.
    – Andreas Blass
    2 days ago

















up vote
0
down vote

favorite












Let $f: C_1 to C_2$ be an order preserving function. Assume that for $A subset C_1$ there exist $Sup(A) in C_1$ and $Sup(f(A)) in C_2$. Prove that $Sup(f(A)) leq f(Sup(A))$.



The statement seems obvious, but how would one go about proving this?










share|cite|improve this question






















  • Show (using the "order-preserving" assumption) that $f(sup(A))$ is an upper bound for $f(A)$. Then use that $sup(f(A))$ is the least upper bound of the same set $f(A)$.
    – Andreas Blass
    2 days ago















up vote
0
down vote

favorite









up vote
0
down vote

favorite











Let $f: C_1 to C_2$ be an order preserving function. Assume that for $A subset C_1$ there exist $Sup(A) in C_1$ and $Sup(f(A)) in C_2$. Prove that $Sup(f(A)) leq f(Sup(A))$.



The statement seems obvious, but how would one go about proving this?










share|cite|improve this question













Let $f: C_1 to C_2$ be an order preserving function. Assume that for $A subset C_1$ there exist $Sup(A) in C_1$ and $Sup(f(A)) in C_2$. Prove that $Sup(f(A)) leq f(Sup(A))$.



The statement seems obvious, but how would one go about proving this?







analysis functions upper-lower-bounds






share|cite|improve this question













share|cite|improve this question











share|cite|improve this question




share|cite|improve this question










asked 2 days ago









user14513462563

846




846












  • Show (using the "order-preserving" assumption) that $f(sup(A))$ is an upper bound for $f(A)$. Then use that $sup(f(A))$ is the least upper bound of the same set $f(A)$.
    – Andreas Blass
    2 days ago




















  • Show (using the "order-preserving" assumption) that $f(sup(A))$ is an upper bound for $f(A)$. Then use that $sup(f(A))$ is the least upper bound of the same set $f(A)$.
    – Andreas Blass
    2 days ago


















Show (using the "order-preserving" assumption) that $f(sup(A))$ is an upper bound for $f(A)$. Then use that $sup(f(A))$ is the least upper bound of the same set $f(A)$.
– Andreas Blass
2 days ago






Show (using the "order-preserving" assumption) that $f(sup(A))$ is an upper bound for $f(A)$. Then use that $sup(f(A))$ is the least upper bound of the same set $f(A)$.
– Andreas Blass
2 days ago

















active

oldest

votes











Your Answer





StackExchange.ifUsing("editor", function () {
return StackExchange.using("mathjaxEditing", function () {
StackExchange.MarkdownEditor.creationCallbacks.add(function (editor, postfix) {
StackExchange.mathjaxEditing.prepareWmdForMathJax(editor, postfix, [["$", "$"], ["\\(","\\)"]]);
});
});
}, "mathjax-editing");

StackExchange.ready(function() {
var channelOptions = {
tags: "".split(" "),
id: "69"
};
initTagRenderer("".split(" "), "".split(" "), channelOptions);

StackExchange.using("externalEditor", function() {
// Have to fire editor after snippets, if snippets enabled
if (StackExchange.settings.snippets.snippetsEnabled) {
StackExchange.using("snippets", function() {
createEditor();
});
}
else {
createEditor();
}
});

function createEditor() {
StackExchange.prepareEditor({
heartbeatType: 'answer',
convertImagesToLinks: true,
noModals: true,
showLowRepImageUploadWarning: true,
reputationToPostImages: 10,
bindNavPrevention: true,
postfix: "",
imageUploader: {
brandingHtml: "Powered by u003ca class="icon-imgur-white" href="https://imgur.com/"u003eu003c/au003e",
contentPolicyHtml: "User contributions licensed under u003ca href="https://creativecommons.org/licenses/by-sa/3.0/"u003ecc by-sa 3.0 with attribution requiredu003c/au003e u003ca href="https://stackoverflow.com/legal/content-policy"u003e(content policy)u003c/au003e",
allowUrls: true
},
noCode: true, onDemand: true,
discardSelector: ".discard-answer"
,immediatelyShowMarkdownHelp:true
});


}
});














 

draft saved


draft discarded


















StackExchange.ready(
function () {
StackExchange.openid.initPostLogin('.new-post-login', 'https%3a%2f%2fmath.stackexchange.com%2fquestions%2f3005689%2fproof-about-ordering-preserving-function%23new-answer', 'question_page');
}
);

Post as a guest















Required, but never shown






























active

oldest

votes













active

oldest

votes









active

oldest

votes






active

oldest

votes
















 

draft saved


draft discarded



















































 


draft saved


draft discarded














StackExchange.ready(
function () {
StackExchange.openid.initPostLogin('.new-post-login', 'https%3a%2f%2fmath.stackexchange.com%2fquestions%2f3005689%2fproof-about-ordering-preserving-function%23new-answer', 'question_page');
}
);

Post as a guest















Required, but never shown





















































Required, but never shown














Required, but never shown












Required, but never shown







Required, but never shown

































Required, but never shown














Required, but never shown












Required, but never shown







Required, but never shown







Popular posts from this blog

Can a sorcerer learn a 5th-level spell early by creating spell slots using the Font of Magic feature?

Does disintegrating a polymorphed enemy still kill it after the 2018 errata?

A Topological Invariant for $pi_3(U(n))$